Elawyers Elawyers
Washington| Change
Find Similar Cases by Filters
You can browse Case Laws by Courts, or by your need.
Find 49 similar cases
AMANUEL WORKU vs FLORIDA ENGINEERS MANAGEMENT CORPORATION, 00-003490 (2000)
Division of Administrative Hearings, Florida Filed:Miami, Florida Aug. 21, 2000 Number: 00-003490 Latest Update: Mar. 23, 2001

The Issue Whether Petitioner is entitled to credit for his answers to questions 42 and 81 of the morning session of the Fundamentals of Engineering Examination portion of the engineering licensure examination given on April 15, 2000.

Findings Of Fact Worku took the Fundamentals of Engineering Examination portion of the examination for licensure to practice as an engineer intern on April 15, 2000. The examination is a national multiple-choice examination developed and administered by the National Council of Examiners for Engineers and Surveyors (NCEES). The examination is divided into a morning session and an afternoon session. The questions in the morning session are worth one raw point each. The questions in the afternoon session are worth two raw points each. Worku challenged questions 42 and 81, which were on the morning session of the examination. Worku received 56 raw points for the morning session and 52 raw points for the afternoon session for a total raw score of 108 on the examination. Based on the NCEES' Score Conversion Table, a raw score of 108 converts to a score of 69. A converted score of 70, which equates to a raw score of 109-113, is a passing score. Question 81 asked the examinee to identify the geometric shape that was given by an equation provided in the question. Each examinee was given a reference manual during the examination. The manual contains general formulas for the types of geometric shapes listed as possible answers to question 81. The equation given in question 81 was for a specific shape and was not listed among the general formulas in the reference manual. Worku felt that because the general equation was not used that the equation was stated incorrectly. However, the equation was stated correctly. The equation differed from the equation listed in the reference manual because it was for a special shape of the geometric figure. Worku did not answer question 81 correctly. Question 42 dealt with recrystallization as it relates to metal. The question asks the examinee to pick the answer which explains the reference to the term "recrystallization" in the question. Worku contends that there are two correct answers to question 42 and that the answer which he provided is one of the correct answers. The answer which Worku provided is not a correct answer. It refers to the process of annealing, which is the process of decreasing the toughness of a metal. Recrystallization can be a part, but is not always part of annealing. Recrystallization and annealing are not synonymous terms; thus Worku is not entitled to credit for question 42.

Recommendation Based on the foregoing Findings of Fact and Conclusions of Law, it is RECOMMENDED that a Final Order be entered finding that Amanuel Worku failed the Engineering Fundamentals Examination with a score of 69. DONE AND ENTERED this 5th day of January, 2001, in Tallahassee, Leon County, Florida. ___________________________________ Susan B. Kirkland Administrative Law Judge Division of Administrative Hearings The DeSoto Building 1230 Apalachee Parkway Tallahassee, Florida 32399-3060 (850) 488-9675 SUNCOM 278-9675 Fax Filing (850) 921-6847 www.doah.state.fl.us Filed with the Clerk of the Division of Administrative Hearings this 5th day of January, 2001. COPIES FURNISHED: Amanuel Worku 18492 Northwest 52nd Path Miami, Florida 33055 Douglas Sunshine, Esquire Florida Engineers Management Corporation 1208 Hays Street Tallahassee, Florida 32301 Natalie A. Lowe, Executive Director Board of Professional Engineers 1208 Hays Street Tallahassee, Florida 32301 Barbara D. Auger, General counsel Department of Business and Professional Regulation 1940 North Monroe Street Tallahassee, Florida 32399-0792

Florida Laws (2) 120.57471.015 Florida Administrative Code (2) 61G15-21.00161G15-21.004
# 1
DENNIS VANN vs DEPARTMENT OF BUSINESS AND PROFESSIONAL REGULATION, FLORIDA ENGINEERS MANAGEMENT CORPORATION, 99-004776 (1999)
Division of Administrative Hearings, Florida Filed:Tampa, Florida Nov. 16, 1999 Number: 99-004776 Latest Update: Jul. 17, 2000

The Issue Whether Petitioner is entitled to additional credit for his responses to Question No. 130 of the Principles and Practice of Engineering portion of the engineering licensure examination administered on April 23, 1999, by the National Council of Examiners for Engineers and Surveyors.

Findings Of Fact Petitioner, Dennis Vann (Petitioner), is an applicant for licensure as a professional engineer in the State of Florida. On April 23, 1999, Petitioner sat for the Principles and Practice Engineering Examination portion of the engineer licensure examinations. This is a national examination developed, controlled, and administered by the National Council of Examiners for Engineering and Surveying (NCEES). Petitioner received a raw score of 45 on this examination. For the electrical engineering discipline, a raw score of 45 results in a converted score of 67. A minimum converted score of 70 is required to pass this examination. A raw score of 48 results in a converted score of 70. Therefore, Petitioner needs an additional 3 raw score points to earn a passing score on the examination. Petitioner challenged the scoring of Question No. 130 on the examination and formally requested the NCEES to rescore his solutions to the question. The NCEES rescored Question No. 130 and determined that Petitioner was not entitled to any additional points for Question No. 130. For Question No. 130, the maximum score achievable was Petitioner received a score of 4 on that item. The NCEES developed and used an Item Specific Scoring Plan (ISSP) for each examination question. Question No. 130 was scored by the NCEES according to the ISSP for that question. Question No. 130 contains two subparts, which require the examinee to address four discrete requirements. Petitioner correctly calculated the bus current (requirement 3). However, Petitioner failed to properly calculate the busway loading and determination of adequacy (requirement 1), the bus impedance (requirement 2), and percent voltage drop (requirement 4). Petitioner's response to Question No. 130 was initially assigned a score of 4. However, if graded correctly, that response would have resulted in a score of 6. The credible testimony of Respondent's expert was that under the ISSP for Question No. 130, Petitioner is entitled to a score of 6 for his response. With a score of 6 for Question No. 130, Petitioner's raw score is increased to 47. A raw score of 47 results in a converted score of 69. Even with the 2 additional points awarded to Petitioner's response to Question No. 130, his score on the professional engineering licensure examination is still below 70 and is not a passing score. Question No. 130 provides all the necessary information for an examinee to solve the problem. Moreover, Question No. 130 is properly designed to test an examinee's competence in electrical engineering.

Recommendation Based upon the foregoing Findings of Fact and Conclusions of Law, it is hereby RECOMMENDED that a final order be entered concluding that Petitioner is entitled to a score of 6 points for his response to Question No. 130, and recalculating Petitioner's total score on the examination on the basis of that conclusion. DONE AND ENTERED this 21st day of April, 2000, in Tallahassee, Leon County, Florida. CAROLYN S. HOLIFIELD Administrative Law Judge Division of Administrative Hearings The DeSoto Building 1230 Apalachee Parkway Tallahassee, Florida 32399-3060 (850) 488-9675 SUNCOM 278-9675 Fax Filing (850) 921-6847 www.doah.state.fl.us Filed with the Clerk of the Division of Administrative Hearings this 21st day of April, 2000. COPIES FURNISHED: Dennis Vann Post Office box 23054 Tampa, Florida 33623 William H. Hollimon, Esquire Ausley & McMullen 227 South Calhoun Street Post Office Box 391 Tallahassee, Florida 32301-1884 Dennis Barton, Executive Director Department of Business and Professional Regulation Board of Professional Engineers 1208 Hays Street Tallahassee, Florida 32301 Natalie A. Lowe, Esquire Florida Engineers Management Corporation 1208 Hays Street Tallahassee, Florida Barbara D. Auger, General Counsel Department of Business and Professional Regulation 1940 North Monroe Street Tallahassee, Florida 32399

Florida Laws (2) 120.569120.57
# 2
OMAR BECKFORD vs FLORIDA ENGINEERS MANAGEMENT CORPORATION, 00-003491 (2000)
Division of Administrative Hearings, Florida Filed:Tallahassee, Florida Aug. 21, 2000 Number: 00-003491 Latest Update: Jan. 19, 2001

The Issue Whether Petitioner is entitled to credit for his answers to Questions 34, 65, and 75 on the Fundamentals of Engineering portion of the engineering licensure examination administered on the morning of April 15, 2000, by the National Council of Examiners for Engineers and Surveyors.

Findings Of Fact Based upon the evidence adduced at hearing and the record as a whole, the following findings of fact are made: On April 15, 2000, as part of his effort to obtain a license to practice as an engineer intern in the State of Florida, Petitioner sat for the Fundamentals of Engineering Examination (Examination). This was a national multiple-choice examination developed and administered by the National Council of Examiners for Engineers and Surveyors (NCEES). The Examination was divided into two sessions: a morning session (AM Part), which tested "lower division subjects" (that is, "the first 90 semester credit hours . . . of engineering course work for a typical bachelor engineering degree program"), and an afternoon session (PM Part), which tested "upper division subjects" (that is, "the remainder of the engineering course work"). Questions on the AM Part were worth one raw point each. Questions on the PM Part were worth two raw points each. The NCEES provided candidates taking the Examination with a Fundamentals of Engineering, Discipline Specific, Reference Handbook (Reference Handbook) that they were allowed to refer to during the Examination. The Reference Handbook, as noted in its Foreword, "contain[ed] only reference formulas and tables; no example problems [we]re included." Petitioner received a total raw score of 104 on the Examination (54 for the AM Part and 50 for the PM Part). According to the NCEES's Score Conversion Table, a raw score of 104 converted to a score of 69. To pass the Examination, a converted score of 70 (or 107-109 raw points) was needed. Accordingly, Petitioner fell three raw points short of receiving a passing score. Petitioner has formally requested that the grading of his answers to Questions 34, 65, and 75 of the AM Part be reviewed. He received no credit for any of these answers. Had these answers been deemed correct (and he received one raw point for each answer), he would have passed the Examination (with a converted score of 70). Question 34 of the AM Part was a clear and unambiguous multiple-choice question that covered subject matter (integral calculus) with which Petitioner and the other candidates should have been familiar. There was only one correct answer to this question, and it was among the responses from which the candidates had to choose. Petitioner chose another answer that was clearly incorrect because it represented a particular solution or expression, and not the "general expression" (representing all solutions) called for by the question. He therefore appropriately received no credit for his answer. Questions 65 and 75 of the AM Part, like Question 34, were clear and unambiguous multiple choice questions that covered subject areas (centroids and thermodynamics, respectively) with which Petitioner and the other candidates should have been familiar. Each of these questions, again like Question 34, had only one correct answer that was listed among the choices from which the candidates had to choose. To answer each question correctly, the candidates had to use a formula that was set forth in the Reference Handbook (on page 21 in the case of Question 65 and on page 46 in the case of Question 75). Petitioner selected neither the correct answer to Question 65, nor the correct answer to Question 75, and therefore was not entitled to any credit for his answers to these questions.

Recommendation Based on the foregoing Findings of Fact and Conclusions of Law, it is RECOMMENDED that a final order be entered rejecting Petitioner's challenge to the failing score he received from the NCEES on the Fundamentals of Engineering portion of the April 15, 2000, engineering licensure examination. DONE AND ENTERED this 15th day of November, 2000, in Tallahassee, Leon County, Florida. STUART M. LERNER Administrative Law Judge Division of Administrative Hearings The DeSoto Building 1230 Apalachee Parkway Tallahassee, Florida 32399-3060 (850) 488-9675 SUNCOM 278-9675 Fax Filing (850) 921-6847 www.doah.state.fl.us Filed with the Clerk of the Division of Administrative Hearings this 15th day of November, 2000.

Florida Laws (5) 120.57455.217471.005471.013471.038 Florida Administrative Code (6) 61-11.01061-11.01261-11.01561-11.01761G15-21.00161G15-21.004
# 3
DON BLACKBURN vs BOARD OF PROFESSIONAL ENGINEERS, 90-005731 (1990)
Division of Administrative Hearings, Florida Filed:Fort Myers, Florida Sep. 10, 1990 Number: 90-005731 Latest Update: Nov. 28, 1990

Findings Of Fact Based upon all of the evidence, the following findings of fact are determined: On April 19, 1990, petitioner, Don R. Blackburn, was a candidate on the engineering intern portion of the professional engineer examination given in Miami, Florida. The test was administered by the Department of Professional Regulation (DPR) on behalf of respondent, Board of Professional Engineers (Board). On July 25, 1990, the Board issued a written uniform grade notice advising petitioner that he had received a grade of 66 on the examination. A grade of 70 is necessary to pass this part of the examination. By letter dated August 15, 1990, petitioner requested a formal hearing to contest his score. In his letter, Blackburn generally contended that the examination was unfairly administered because certain books were allowed to be used by some but not all candidates, untrained proctors were given the authority to scan review materials and determine which could or could not be used by the candidates, and because of the chaos and confusion that occurred during the examination, he was unable to attain a score that he otherwise would have been able to achieve. Blackburn is an engineer for Lee County and is seeking to pass the engineering intern portion of the examination. A passing grade on that portion is a prerequisite to sitting on the second part of the professional engineer examination. He has taken the examination on a number of occasions and has gradually improved his score to just short of passing. Indeed, on the October 1989 examination, Blackburn scored a 69, or just one point less than the required 70. Prior to the April 1990 examination, the engineering intern portion of the professional engineer examination was an unrestricted open book examination. This meant candidates could use any and all reference and review materials during the examination. Beginning with the April 1990 examination, the Board imposed certain restrictions on the use of review materials. As early as October 9, 1989, the Board's executive director sent a memorandum to all candidates on the October 1989 examination, including Blackburn, concerning the new restrictions. The memorandum stated in part: Please be advised of certain restrictions listed in the Candidate Information Booklet which will not be implemented until the April 1990 examination. These restrictions are found in the "Examination Administration Information" section and are concerning the following two areas: * * * 2. Books or information containing sample questions or engineering problems may also be brought provided they are bound. Again, the new restrictions listed in the Candidate Information Booklet regarding the above two areas WILL NOT be implemented until the April 1990 examination. All candidates on the April 1990 examination were given a Candidate Information Booklet prepared in January 1990 by DPR's Bureau of Examination Services. On pages 13 and 14 of the booklet was found the following information: This is an open book examination. Candidates may use textbooks, handbooks, notes, and reference materials which are bound, copyrighted and printed. The term "bound" refers to material that is bound permanently, hard or paperback stitched or glued, or spiral, plastic or three-ringed bound. The printed material must remain contained (bound) in its cover during the entire examination. No writing tablets, unbound tablets or unbound "loose notes" will be allowed. No books with contents directed toward sample questions or solutions of engineering problems are permitted in the examination room. Examinees are not permitted to exchange reference materials or aids during the examination. (Emphasis in original) What the emphasized language meant is that "review" manuals, which contain problems and solutions, were prohibited from use during the examination while "reference" books were not. However, the booklet did not list the specific names of published materials that would be permitted or excluded. In order to ascertain which books he might use on the next examination, on March 27, 1990, Blackburn telephoned the Board in Tallahassee and spoke with a female employee named "B. J." who advised him that "review publications directed principally towards the solution of engineering problems" would be excluded. When asked if "Lindeburg's Sixth Edition" would be authorized, B. J. told Blackburn she wasn't sure and that it would be left up to the proctors in the room. She did say, however, that a review manual authored by Schaum could be used. The engineering intern examination in April 1990 was administered in two separate rooms at the Radisson Hotel in Miami, Florida. Blackburn was in a "very large" upstairs room with approximately thirty other candidates while a similar number took the examination in a downstairs room. The examination in the upstairs room began at 8:43 a.m. after various instructions were read to the candidates by the examination supervisor, Jeannie Smith, a veteran of twenty years in proctoring and supervising professional examinations. According to Smith, there was "considerable confusion" concerning which books could be used by the candidates, particularly since this was the first examination given with the new restrictions. She also acknowledged that there was "chaos" prior to the beginning of the examination and that this was, "extremely upsetting" to the examinees. However, before the examination began, Smith announced on a microphone the names of certain books which the Board had given her that were either prohibited or could be used by candidates. She further advised that if candidates had any questions they were to come to a bulletin board by the microphone where she had posted Xerox copies of the covers of various books. If a book could be used, it had the word "YES" printed on the cover while a "NO" was printed on those covers of books that could not be used. 1/ It is noted that only one cover sheet with a "YES" was posted, that being the Civil Engineering Reference Manual, Fourth Edition, Michael R. Lindeburg. However, at least three candidates who took the examination that morning, including petitioner, did not see the posted materials nor hear the invitation for candidates to come to the bulletin board. One book in issue that was specifically prohibited was Engineer In Training Review Manual, Sixth Edition, Michael R. Lindeburg, which contained 378 solved problems, and thus fell within the general prohibition of review manuals described on page 14 of the Candidate Information Booklet. However, those candidates who had the Seventh Edition of the same book were allowed to keep and use that manual even though it contained 422 solved problems, or some 44 more solved problems than was contained in the prohibited Sixth Edition. By allowing those students having the Seventh Edition to use the same even though it contained "review" materials, DPR violated the instructions contained in the Candidate Information Booklet and gave an advantage to those candidates not enjoyed by others, including petitioner. In addition, at least one other candidate in the upstairs group was allowed to use a prohibited review manual (Schaum's Outline Series, Theory and Problems of Electric Power Systems) but still that candidate did not attain a passing grade. Petitioner also contended that candidates taking the examination in the downstairs room were allowed to use language dictionaries during the morning part of the examination while those upstairs could not. 2/ Petitioner's contention is grounded upon hearsay evidence and accordingly it is found that no competent proof to support this claim was submitted. However, there was obviously some confusion over this matter because, after receiving complaints of this nature from two candidates, Smith telephoned the Board's offices in Tallahassee during the lunch break to ascertain whether such books could be used. Upon learning that they could not, she advised the upstairs group at the beginning of the afternoon session that dictionaries were not allowed. Blackburn also established that during the examination proctors went from desk to desk examining the materials that each candidate had in his possession. If a candidate had what the proctor perceived to be a book containing solutions to problems, the candidate was told to put the book on the floor. In the alternative, she candidates were told that if they tore the offending pages out of the book, they could continue using the remaining materials. Petitioner has complained that the proctors were not engineers and they were untrained in determining whether a book was acceptable or not. The Board has conceded that engineers do not proctor examinations but asserted that they are intelligent enough to determine whether books fall within the proscribed category. According to Blackburn's proctor at the examination, George Walton, a retired Coast Guard captain and engineering graduate of the Coast Guard Academy, he relied upon the list of approved and disapproved books supplied by the Board prior to the examination in determining whether materials would be excluded or not. Walton also stated that if he examined a book and found it contained solutions, he would disallow the same unless the offending pages were removed. A DPR expert in testing and measurements, Dr. Joseph A. Klock, examined the pass/fail rate for the examination taken by Blackburn and compared that rate to the October 1989 examination rate. Doctor Klock found no significant difference in the two rates and concluded that there was no statistically significant difference in performance of candidates over those time periods despite the confusion which occurred during the April 1990 examination. Blackburn did not present any evidence to show that if he had used the Seventh Edition of the Engineer In Training Review Manual, he would have been able to achieve more points on a particular problem and thus would have had a passing grade. Blackburn's principal complaint was that he had spent many hours preparing for the examination in question, that he was forced to guess which books to bring to the examination, and because of the confusion and chaos that took place at the beginning of the examination as well as his awareness that others were using a review manual with solved problems, it was impossible for him to give his best effort on the examination.

Recommendation Based on the foregoing findings of fact and conclusions of law, it is RECOMMENDED that petitioner's request to receive a passing grade on the April 1990 professional engineer examination be DENIED. However, petitioner should be entitled to retake the next examination at no charge. DONE and ENTERED this 28th day of November, 1990, in Tallahassee, Florida. DONALD R. ALEXANDER Hearing Officer Division of Administrative Hearings The DeSoto Building 1230 Apalachee Parkway Tallahassee, FL 32399-1550 (904) 488-9675 Filed with the Clerk of the Division of Administrative Hearings this 28th day of November, 1990.

Florida Laws (1) 120.57
# 4
CARLOS MARTINEZ MALLEN vs BOARD OF PROFESSIONAL ENGINEERS, 89-005973 (1989)
Division of Administrative Hearings, Florida Filed:Miami Beach, Florida Nov. 01, 1989 Number: 89-005973 Latest Update: Mar. 28, 1990

Findings Of Fact Petitioner, Carlos Martinez Mallen, is an applicant for licensure by endorsement to become a professional engineer in the State of Florida. He filed his application for licensure with the Florida Board of Professional Engineers (hereinafter "Board") in January 1988, relying on the facts that he was licensed in Spain approximately 25 years ago and has approximately 30 years of experience as a professional engineer. The Board subsequently determined that he could not be considered for licensure by endorsement. Petitioner has never taken a licensing examination in the United States which is substantially equivalent to the examination required for licensure by Section 471.013, Florida Statutes, and described in Chapter 21H, Florida Administrative Code. Further, Petitioner has never been licensed in any state or territory of the United States, although he does hold a license to practice engineering in Spain. On the other hand, Petitioner's engineering experience record shows that he has considerable experience in the practice of engineering which would meet the additional experience requirements of Section 471.013, Florida Statutes. The Board, having determined that Petitioner does not qualify for licensure by endorsement, performed an analysis of Petitioner's application to determine whether his degree from the University of Madrid was an engineering degree which might qualify him to sit for the 1icensure examination and to ascertain if Petitioner could obtain licensure by that alternative method. An analysis was made by the Board's Education Advisory Committee to determine whether the curriculum for Petitioner's degree from the University of Madrid met the requirements of Rule 21H-20.006, Florida Administrative Code. This analysis was specifically directed to determine whether Petitioner's curriculum conformed to the criteria for accrediting engineering programs set forth by the Engineering Accreditation Commission of the Accreditation Board of Engineering and Technology, Inc., (hereinafter "ABET"). The analysis of Petitioner's degree shows that, when compared with ABET criteria, Petitioner's engineering education was deficient four semester hours in mathematics and included no courses in engineering design, sixteen semester hours of which are required by ABET criteria. Further, Petitioner's education included no computer application of engineering design programs, a mandated requirement by ABET standards. Petitioner has never taken any of these courses subsequent to receiving his degree in Spain. Petitioner's degree, rather than being an engineering degree, is the equivalent of a bachelor's degree in chemistry. Petitioner's degree is significantly deficient in required course areas, so that it does not meet the Board's criteria. Petitioner thus cannot be considered as an applicant for examination since in order to sit for the professional engineer examination in the State of Florida, one must have an engineering degree which meets standards acceptable to the Board. Finally, Petitioner's background was reviewed to determine whether he could be considered for licensure under a different provision for licensure by endorsement. Petitioner has never held a professional engineer registration or license from another State of the United States. The Board has never interpreted the word "state" found in the statutes and rules regulating the licensure of professional engineers in Florida to include foreign counties. Petitioner is not a graduate of the State University System. Petitioner did not notify the Department before July 1, 1984, that he was engaged in engineering work on July 1, 1981, and wished to take advantage of a temporary educational waiver. As a result of the Board's review of all avenues to licensure available to Petitioner, Petitioner's application was denied either to sit for the examination to become a professional engineer or to be licensed by endorsement, unless and until he meets the educational requirements to sit for the professional engineer examination.

Recommendation Based upon the foregoing Findings of Fact and Conclusions of Law, it is RECOMMENDED that a Final Order be entered denying Petitioner's application for licensure by endorsement and further finding that Petitioner's educational background does not meet the requirements necessary to take the examination to become licensed in the State of Florida. DONE AND ENTERED in Tallahassee, Leon County, Florida, this 28th day of March, 1990. LINDA M. RIGOT Hearing Officer Division of Administrative Hearings The DeSoto Building 1230 Apalachee Parkway Tallahassee, Florida 32399-1550 (904) 488-9675 Filed with the Clerk of the Division of Administrative Hearings this 28th day of March, 1990. APPENDIX TO RECOMMENDED ORDER, CASE NO. 89-5973 Petitioner's proposed paragraphs numbered 0.00, .10, .20, .30, .40, .50, 1.10, 1.20, 2.20, 3.10, 3.20, 3.40, 3.60, 4.10, 4.11, 4.13, 5.00, 5.30, 5.40, 5.41, 5.50, 5.51, 5.52, 6.00, 6.10, 6.20, 6.21, 6.22, 6.23, 6.24, 6.25, 6.26, 7.00, 7.40, and 7.50 have been rejected as not constituting findings of fact but rather as constituting argument or conclusions of law. Petitioner's proposed paragraphs numbered 1.21, 3.00, 4.00, 7.10, 7.20, 730, 7.41, 7.42, and 7.43 have been rejected as being contrary to the weight of the evidence in this cause. Petitioner's proposed paragraphs numbered 1.22 and 2.10 have been adopted either verbatim or in substance in this Recommended Order. Petitioner's proposed paragraphs numbered 3.30, 3.50, 3.70, 4.12, 4.20, 5.10, 5.11, and 5.20 have been rejected as being irrelevant to the issues involved in this proceeding. Respondent's proposed findings of fact numbered 1-8 have been adopted either verbatim or in substance in this Recommended Order. COPIES FURNISHED: John J. Rimes, III, Esquire Office of Attorney General Department of Legal Affairs The Capitol Tallahassee, Florida 32399-1050 Carlos Martinez Mallen 33C Venetian Way #66 Miami Beach, Florida 33139 Kenneth E. Easley, General Counsel Department of Professional Regulation 1940 North Monroe Street Tallahassee, Florida 32399-0792 Rex Smith, Executive Director Department of Professional Regulation Board of Professional Engineers 1940 North Monroe Street Tallahassee, Florida 32399-0792

Florida Laws (9) 120.57471.005471.013471.0156.107.207.417.437.50
# 5
BOARD OF ACCOUNTANCY vs. GARY L. WHEELER, 79-002310 (1979)
Division of Administrative Hearings, Florida Number: 79-002310 Latest Update: Mar. 26, 1980

Findings Of Fact Based upon my observation of the witnesses and their demeanor while testifying, the arguments of counsel and the entire record compiled herein, the following relevant facts are found. Gary L. Wheeler, Respondent, is a graduate of Bob Jones University, having received a Bachelor of Science degree therefrom in accounting in 1974. On July 27, 1979, Respondent received his California certificate as a certified public accountant. Thereafter, Respondent filed an application to obtain a reciprocal C.P.A. certificate in Florida based on his certificate issued by the State of California (Certificate No. E-28234). His application was denied by the Petitioner on October 26, 1979, for the following reason: Applicant failed to satisfy the requirements set forth in Section 7(3)(b), Chapter 79-202, Laws of Florida, inasmuch as the license issued to Gary L. Wheeler in California is not issued under criteria substantially equivalent to that in effect in Florida at the time the California license was issued. Bob Jones University was not recognized as an accredited university in Florida by the Board when Respondent received his California certificate inasmuch as it was not listed among the institutions of postsecondary education by the Council on Postsecondary Accreditation (COPA). During September, 1976, Petitioner adopted the COPA list of schools as the schools from which it would accept graduates to sit for its examination. This was done for the avowed purpose of ensuring minimum competence and technical fitness among the ranks of Florida accountants. Douglas H. Thompson, Jr., the Petitioner's Executive Director since 1968, is the Board's chief operating officer and carries out its functions respecting applications for licensure. As such, Mr. Thompson was the person charged with examining Respondent's application pursuant to his California certificate to determine whether the Respondent's certificate was issued under criteria "substantially equivalent" to Florida's licensing criteria. Respondent's application was considered by the Board on two (2) occasions and rejected because Respondent's alma mater, Bob Jones University, is not listed among the accredited schools and universities by COPA. See Sections 473.306; 473.307 and 473.308, Florida Statutes, as amended; and Chapter 21A-28.06, Florida Administrative Code. As an aside, it was noted that the Board, in adopting its procedure for evaluating the criteria for applicants who were seeking to obtain certificates based on the reciprocal qualifications guidelines also adopted other equivalency procedures which provide Respondent an alternative method for which he may obtain a Florida certificate. In this regard, Respondent is only approximately six (6) quarter hours away from obtaining his certificate under the alternative equivalency procedures established by the Board. See Chapters 21A-9.01 through 9.04(4), Florida Administrative Code.

Recommendation Based on the foregoing Findings of Fact and Conclusions of Law, it is hereby RECOMMENDED that Respondent's appeal of the Board's action in denying his application for a reciprocal license to practice public accounting based on the issuance of his California certificate be DENIED. DONE AND ORDERED in Tallahassee, Leon County, Florida, this 26th day of March, 1980. JAMES E. BRADWELL Hearing Officer Division of Administrative Hearings Oakland Building 2009 Apalachee Parkway Tallahassee, Florida 32301 (904) 488-9675

Florida Laws (3) 120.57473.306473.308
# 6
YEVGENIYA G. SOKOL vs BOARD OF PROFESSIONAL ENGINEERS, 97-001760 (1997)
Division of Administrative Hearings, Florida Filed:Tampa, Florida Mar. 31, 1997 Number: 97-001760 Latest Update: Mar. 16, 1998

The Issue Whether Petitioner is eligible for licensure by endorsement as a professional engineer and/or waiver of Part I of the engineering licensing examination.

Findings Of Fact From 1969 through 1974, Petitioner attended the Lipetsk Branch of the Moscow Institute of Steel and Alloys. In 1974, Petitioner graduated from the Lipetsk Polytechnical Institute (Institute) in Russia, with a degree in industrial and civil engineering. The degree in civil engineering earned by Petitioner is equivalent to a bachelor of science degree in civil engineering in the United States. Upon graduation from the Institute, Petitioner commenced her professional employment as an engineer on August 30, 1974, at the LIPTSKGRAZHDANPROEKT Design Institute. In January 1976, Petitioner was employed as a professional engineer at the State Design Institute DNEPRPROEKKTSTALKONSTRUKSIYA (DNEPR), where she was continuously employed until leaving the Soviet Union in December 1993. There are no specific licensing or registration requirements in Russia with respect to engineers. Therefore, after earning a degree in industrial and civil engineering, Petitioner could work as an engineer in Russia without taking any professional examination. However, in 1979, Petitioner took an examination in Russia that allowed her to sign her own drawings and calculations. During Petitioner's tenure at the DNEPR, she achieved the status of Senior Engineer in 1986; was promoted to the position of Category II Engineer for Steel Structures in 1988; was promoted to the post of Category I Engineer for Steel Structures in 1990; and was elevated to the position of Leading Engineer in 1991. To achieve the status of Category II Engineer for Steel Structures and Category I Engineer for Steel Structures at DNEPR, Petitioner had to take an examination in 1988 and in 1990, respectively. The promotion to each of these positions was predicated upon Petitioner's passing these examinations and demonstrating expertise in the areas of economics, chemistry, mathematics, physics, building materials, corrosion prevention, resistance of materials, and construction mechanics. As a result of passing the examinations in 1988 and 1990, Petitioner was not only promoted, but also received salary increases. Petitioner believes that the two examinations she took in Russia in 1988 and in 1990, while working at the DNEPR were substantially equivalent to the Fundamentals Examination. However, no evidence was presented to support this claim. The Fundamentals Examination is one component of the engineering licensing examination, and is designed to assess whether an individual is qualified to practice in this state as an engineer intern. This examination is usually taken either in the applicant's last year in engineering school or shortly after graduation. With regard to format, the Fundamentals Examination is an eight-hour examination and consists of 120 multiple-choice questions. The Principles and Practice Examination is the second part of the engineering licensing examination and is taken after successful completion of the Fundamentals Examination. Oscar E. Olsen, a structural engineer and owner of O.E. Olsen and Associates, a structural engineering firm, is currently Petitioner's employer. Mr. Olsen, who is generally familiar with the Fundamentals Examination, testified that the list of subjects covered on the two examinations taken by Petitioner in 1988 and 1990, coincide with the subject matter on the Fundamentals Examination. Mr. Olsen further testified that it appeared to him that the two examinations taken by Petitioner were comparable to the Fundamental Examinations required in Florida. Notwithstanding his testimony that the exams taken by Petitioner are substantially equivalent to the Fundamentals Examination, Mr. Olsen admitted that he has never seen or reviewed the examinations taken by Petitioner while she was in Russia. It is impossible to render a reasonable opinion as to whether the two examinations taken by Petitioner in Russia are substantially equivalent to the Fundamentals Examination, where the only information provided with regard to the former is a list of subject areas covered. Such a list gives no indication of the depth and specific content of the subject matter on the examinations; the difficulty of the examinations; the passing scores; the number and format of the questions; and the length of the examinations.

Recommendation Based on the foregoing Findings of Fact and Conclusions of Law, it is recommended that Respondent, the Board of Professional Engineers, enter a Final Order denying Petitioner's request for waiver of Part I, the Fundamentals Examination, and for licensure by endorsement. DONE AND ENTERED this 9th day of December, 1997, in Tallahassee, Leon County, Florida. CAROLYN S. HOLIFIELD Administrative Law Judge Division of Administrative Hearings The DeSoto Building 1230 Apalachee Parkway Tallahassee, Florida 32399-3060 (904) 488-9675 SUNCOM 278-9675 Fax Filing (904) 921-6847 Filed with the Clerk of the Division of Administrative Hearings this 9th day of December, 1997. COPIES FURNISHED: Murray Silverstein, Esquire Powell, Carney, Hayes, and Silverstein One Plaza, Suite 1210 St. Petersburg, Florida 33731-1689 Edwin A. Bayo Assistant Attorney General Office of the Attorney General The Capitol, Plaza 01 Tallahassee, Florida 32399-1050 Angel Gonzalez Executive Director Board of Professional Engineers Northwood Centre 1940 North Monroe Street Tallahassee, Florida 32399-0755 Lynda L. Goodgame, General Counsel Department of Business and Professional Regulation 1940 North Monroe Street Tallahassee, Florida 32399-0792

Florida Laws (5) 120.57212.06471.008471.013471.015
# 7
MARIANITO MANALO ILAGAN vs. BOARD OF ACCOUNTANCY, 80-000210 (1980)
Division of Administrative Hearings, Florida Number: 80-000210 Latest Update: Jul. 11, 1980

Findings Of Fact Petitioner is A graduate of the University of the East in Manila, Philippines. Petitioner is the holder of the state of Illinois C.P.A. Certificate No. 18012. On July 11, 1979, Petitioner filed an application to obtain a reciprocal certified public accountant certificate in Florida (licensure by endorsement) based upon his certificate issued by the State of Illinois. On December 14, 1979, the Board denied Petitioner's application for a reciprocal certificate for the reason that Petitioner had not graduated from an accredited four-year college or university and, accordingly, failed to satisfy the requirements set forth in Section 7(3)(b), Chapter 79-202, Laws of Florida, now codified as Section 473.308(3)(b), Florida Statutes (1979) The University of the East in Manila, Philippines, is not recognized by the Board as an accredited university in Florida and was not so recognized at the time that Petitioner received his certificate as a certified public accountant in the State of Illinois. The University of the East is not listed among the institutions of post secondary education by the Council on Postsecondary Accreditation, the official listing of accredited colleges and universities adopted by the Board to ensure the minimum competence of public accounting practitioners. Additionally, the University of the East in Manila, Philippines, has not been accredited by any of the regional accrediting agencies recognized by the Board. Douglas H. Thompson, Jr., the Respondent's Executive Director since 1968, is the Board's chief executive officer and, as such, carries out the Board's functions respecting applications for licensure. Mr. Thompson examined Petitioner's application pursuant to Petitioner's Illinois certificate to ascertain whether Petitioner's certificate was issued under criteria substantially equivalent to Florida's licensing criteria and determined that the criteria were not substantially equivalent. Petitioner's application was considered by the Board on two occasions and was rejected.

Recommendation Based upon the foregoing findings of fact and conclusions of law, it is RECOMMENDED THAT: Petitioner's application for a reciprocal certified public accountant certificate be denied. RECOMMENDED this 10th day of June, 1980, in Tallahassee, Florida. LINDA M. RIGOT Hearing Officer Division of Administrative Hearings Collins Building Room 101 Tallahassee, Florida 32301 (904) 488-9675 Filed with the Clerk of the Division of Administrative Hearings this 10th day of June, 1980. COPIES FURNISHED: Samuel Hankin, Esquire Commerce Building 226 South Main Street Gainesville, Florida 32602 Mr. Marianito Manalo Ilagan 9020 S.W. 56th Street Cooper City, Florida 33328 Ms. Nancy Kelley Wittenberg Secretary Department of Professional Regulation The Oakland Building 2009 Apalachee Parkway Tallahassee, Florida 32301

Florida Laws (3) 120.57473.306473.308
# 8
MAGDALENA COSTIN vs FLORIDA ENGINEERS MANAGEMENT CORPORATION, 98-002584 (1998)
Division of Administrative Hearings, Florida Filed:Jacksonville, Florida Jun. 05, 1998 Number: 98-002584 Latest Update: Feb. 23, 1999

The Issue The issue to be resolved is whether Petitioner is entitled to additional credit for her response to question nos. 122 and 222 of the civil engineering examination administered on October 31, 1997.

Findings Of Fact On October 31, 1997, Petitioner took the civil professional engineering licensing examination. A score of 70 is required to pass the test. Petitioner obtained a score of 69. Petitioner challenged the scoring of question nos. 122 and 222. As part of the examination challenge process, Petitioner's examination was returned to the National Council of Examiners for Engineering and Surveying where it was re-scored. In the re-score process, the grader deducted points from Petitioner's original score. Petitioner was given the same raw score of 6 on question number 122; however, on question number 222 her raw score of 4 was reduced to a 2. Petitioner needed a raw score of 48 in order to achieve a passing score of 70; she needed at least three additional raw score points to obtain a passing raw score of 48. Petitioner is entitled to a score of 6 on problem number 122. The solution and scoring plan for that problem required the candidate to obtain a culvert size in the range of 21-36 inches. The Petitioner incorrectly answered 3.1 feet or 37.2 inches. She is not entitled to additional credit for problem number 122 because she answered the question with the wrong size culvert. Problem number 122 required the candidate to use a predevelopment peak flow of 40 cubic feet per second (cfs). Petitioner used 58.33 cfs. She chose the maximum flow rather than the predevelopment peak flow. In solving problem number 122, Petitioner chose a design headwater depth of 4.8 feet. The correct solution required a design headwater depth of 5.7 feet. Petitioner made another mistake in problem number 122; she failed to check the water depth in the downstream swale. Petitioner concedes she was given sufficient information to solve problem number 122. She understood what the question was asking of her. She admits that she did not compute the critical depth of the water and that she did not complete the solution. Question number 222 had three parts. The candidate was required to determine the footing size, to select the reinforcing steel, and to provide a sketch for a concrete column located along the edge of a building. Petitioner understood the question and was provided enough information to solve the problem. Petitioner correctly checked the footing size as required by the first part; however, she did not select the reinforcing steel or show the required sketch. Therefore, Petitioner did not complete enough of the problem to qualify for a score of 4 points. She is entitled to a score of 2 points. The examination questions at issue here were properly designed to test the candidate's competency in solving typical problems in real life. The grader (re-scorer) utilized the scoring plan correctly. Petitioner has been in the United States for approximately eleven years. She lived in Romania before she came to the United States. In Romania, Petitioner used only the metric system in her professional work. While she has used the English system since moving to the United States, Petitioner is more familiar with the metric system. The Principles and Practice examination is an open-book examination. Petitioner took a book entitled the Fundamentals of Engineering Reference Handbook to the examination. When the proctor examined her books, she told the Petitioner she was not permitted to keep the handbook. The proctor took the handbook from the Petitioner. Petitioner protested the confiscation of her reference book because she had used the same book in two previous tests. About ten minutes later, the proctor's supervisor returned the book to Petitioner. Petitioner's book was returned at least ten minutes before the test began. She was permitted to use the book during the test. There is no persuasive evidence that the proctor's mistake in temporarily removing Petitioner's reference book caused her to be so upset that she failed the test. Candidates were not permitted to study their books prior to the beginning of the examination. Petitioner may have been nervous when the test began. However, Petitioner received a perfect score of ten points on the first problem she worked, problem number 121.

Recommendation Based upon the findings of fact and conclusions of law, it is RECOMMENDED that the Board of Professional Engineers enter a Final Order confirming Petitioner's score on the examination and dismissing the Petitioner's challenge. DONE AND ENTERED this 13th day of January, 1999, in Tallahassee, Leon County, Florida. SUZANNE F. HOOD Administrative Law Judge Division of Administrative Hearings The DeSoto Building 1230 Apalachee Parkway Tallahassee, Florida 32399-3060 (850) 488-9675 SUNCOM 278-9675 Fax Filing (850) 921-6847 www.doah.state.fl.us Filed with the Clerk of the Division of Administrative Hearings this 13th day of January, 1999. COPIES FURNISHED: Natalie A. Lowe, Esquire Board of Professional Engineers 1208 Hays Street Tallahassee, Florida 32301 William Bruce Muench, Esquire 438 East Monroe Street Jacksonville, Florida 32202 Lynda L. Goodgame, General Counsel Department of Business and Professional Regulation 1940 North Monroe Street Tallahassee, Florida 32399-0792 Dennis Bartin, President Florida Engineers Management Corporation 1940 North Monroe Street Tallahassee, Florida 32399-0792

Florida Laws (1) 120.57
# 9
JONATHAN A. BATISTA vs BOARD OF PROFESSIONAL ENGINEERS, 20-003075RX (2020)
Division of Administrative Hearings, Florida Filed:Tallahassee, Florida Jul. 10, 2020 Number: 20-003075RX Latest Update: Jun. 29, 2024

The Issue The issue in this case is whether Florida Administrative Code Rule 61G15-21.004(2) is an invalid exercise of delegated legislative authority.

Findings Of Fact The Parties Petitioner is an applicant for licensure as a professional engineer ("P.E.")2 in Florida, and currently works in the discipline of environmental engineering in Florida. His practice focuses primarily on water-related areas within that discipline. Petitioner is not currently licensed as a P.E. Respondent is a board within the Department of Business and Professional Regulation ("Department"). It is the state agency created pursuant to section 471.007, Florida Statutes, and charged with licensing professional engineers in Florida. Respondent is vested with the authority to adopt rules to implement chapter 471, regarding the regulation of the practice of engineering in Florida, as defined in section 471.005(7). Respondent adopted the Challenged Rule at issue in this proceeding. Statutory and Rule Background The engineering profession in Florida is regulated pursuant to chapter 471. A person may become licensed as a P.E. in Florida by applying for licensure, fulfilling specified educational and experience requirements, and 2 For purposes of this Final Order, the terms "professional engineer" or "P.E." will be used to refer to persons who are licensed engineers under chapter 471, Florida Statutes. either being endorsed for licensure as provided in sections 471.015(3) and (5), or passing the required licensure examinations. § 471.015, Fla. Stat. Pursuant to section 471.015, Respondent has adopted Florida Administrative Code Chapter 61G15-20, which codifies, in rule, the requirements for licensure as a P.E. in Florida. An applicant for licensure must be a graduate of a Board-approved engineering program; have the requisite number of years of engineering experience; and have passed the specified licensure exams. Fla. Admin. Code R. 61G15-20.0010. Section 455.217(1)(d) authorizes Respondent to adopt, by rule, the use of a national professional licensing examination that the Department has certified as meeting requirements of national examinations and generally accepted testing standards. To implement section 455.217(1), Respondent has adopted rule 61G15- 21.001, titled "Examination Designated; General Requirements." This rule requires that, unless an applicant qualifies for licensure by endorsement, he or she must pass the National Council of Examiners for Engineering and Surveyors ("NCEES") licensure exam. Part I of the NCEES exam is the Fundamentals of Engineering ("FE") exam, and part II of the NCEES exam is the Principles and Practice Exam ("PP") exam. Respondent has entered into a contract with NCEES to provide the FE and PP exams in Florida. A person must pass both the FE and PP exams to be licensed as a P.E. in Florida. § 471.015(1), Fla. Stat. The Challenged Rule states: "[t]he passing grade for Principles and Practice Exam is determined by the National Council of Examiners for Engineering and Surveying, where psychometric statistical methods are used to determine the level of performance that corresponds with minimal competence in the discipline." Fla. Admin. Code R. 61G15-21.004(2). The Challenged Rule is a subsection of rule 61G15-21.004, which is titled "Passing Grade." The Challenged Rule specifically and exclusively addresses the method for determining the passing grade on the PP exam. Sections 455.217(1)(d) and 471.013 are cited as the rulemaking authority for the Challenged Rule, and sections 455.217(1)(d) and 471.015(1) are cited as the law implemented by the Challenged Rule. The term "engineering," as used in section 471.005(7), includes the term "professional engineering," and defines the types of services and creative work that constitutes "engineering." An "engineer," as defined in section 471.005(5), includes the terms "professional engineer" and "licensed engineer," and means a person who is licensed to engage in the practice of engineering under chapter 471. By contrast, an "engineer intern," as defined in section 471.005(6), means a person who has graduated from a Board-approved engineering curriculum and has passed the FE exam. By definition, these are distinct terms. The term "engineer" is used to describe a person licensed as a P.E. under chapter 471, while the term "engineer intern" is used to described a person who may engage in the kinds of activities described within the term "engineering," as defined in section 471.005(7), but who is not licensed as a P.E. in Florida, and, therefore, is not authorized to hold himself or herself out as a licensed engineer in Florida. The Rule Challenge Petition The Rule Challenge Petition alleges four grounds under section 120.52(8) for invalidating the Challenged Rule. Alleged Invalidity of Challenged Rule under Section 120.52(8)(c) In paragraph 15 of the Rule Challenge Petition, Petitioner asserts that section 455.217(1)(c)—which, at the time the Rule Challenge Petition was filed, was cited as the specific authority for, and law implemented by, the Challenged Rule—did not authorize the Challenged Rule, so that the Challenged Rule enlarged, modified, or contravened the specific provisions of law implemented, pursuant to section 120.52(8)(c). As a result of Petitioner having filed the Rule Challenge Petition, Respondent discovered that it had not updated its citation of the specific authority for, and law implemented by, the Challenged Rule, when section 455.217(1) was amended and renumbered in 1997, so that section 455.217(1)(c) no longer was the correct citation to the law implemented by the Challenged Rule. Respondent requested the Department of State, Administrative Code and Register Section ("DOS"), to make a technical, non-substantive change to the Challenged Rule. As authorized by Florida Administrative Code Rule 1-1.010(10), DOS updated the statutory citation to section 471.217(1)(d), which is the correct citation to the law implemented by the Challenged Rule. This technical change nullifies the alleged invalidity ground set forth in paragraph 15 of the Rule Challenge Petition, and Petitioner concedes this. In paragraph 16 of the Rule Challenge Petition, Petitioner also alleges that the Challenged Rule enlarges, modifies, or contravenes section 455.217(1)(a), because the PP examination does not adequately and reliably measure an applicant's ability to practice the profession regulated by the Department. However, as discussed below, section 455.217(1)(a) is not cited as a specific provision of law implemented by the Challenged Rule, so cannot form the basis of a challenge to the Rule under section 120.52(8)(c). Alleged Invalidity of Challenged Rule under Section 120.52(8)(d) In paragraph 17 of the Rule Challenge Petition, Petitioner alleges that the Challenged Rule is invalid under section 120.52(8)(d) because it is vague, fails to establish adequate standards for agency decisions, or vests unbridled discretion in the agency. In support of this alleged invalidity ground, Petitioner asserts that the Challenged Rule is vague because "the level of performance on the PP exam is stated to correspond with minimal competency, yet there are no rules which provide definitive guidance to NCEES on what constitutes the general areas of competency in regards to engineering practice."3 Petitioner also alleges, in paragraph 19 of the Rule Challenge Petition, that the Challenged Rule is invalid under section 120.52(8)(d) because it equates passage of the PP exam with a 30-year practice experience requirement for licensure by endorsement set forth in section 471.015(5)(b). To this point, Petitioner states: "I can't think of anything more arbitrary than the principles and practice exam equating to near[-]retirement level experience."4 Alleged Invalidity of Challenged Rule under Section 120.52(8)(e) In paragraph 18 of the Rule Challenge Petition, Petitioner alleges that the Challenged Rule is invalid pursuant to section 120.52(8)(e) because it is arbitrary, for several reasons. Paraphrased, these reasons are: passage of the PP exam does not accurately reflect, or equate to, minimal competence in the discipline; the PP Exam does not accurately evaluate an individual's engineering ability level, but instead evaluates an individual's exam performance compared to average group exam performance; the PP exam does not reliably distinguish between minimal competence and incompetence to practice engineering, as evidenced by the fact that engineers who fail the PP exam still competently perform, and, thus, keep, their engineering jobs; passing the PP exam, by itself, does not certify an individual to competently perform any engineering service or creative work as defined in 3 Section 455.217(1)(b) requires, for each exam developed by the Department or a contracted vendor, that the general areas of competency covered by the exam be specified by rule. The last sentence of that subsection states that the requirements of subsection (b) do not apply to national exams, such as the NCEES PP exam, which are approved and administered pursuant to section 455.217(1)(d). Thus, the law implemented by the Challenged Rule does not require areas of competency to be specified in the Challenged Rule. 4 Because paragraph 19 of the Rule Challenge Petition alleges that the rule is arbitrary, the undersigned considers this paragraph to constitute a challenge to the Challenged Rule under section 120.52(8)(e), rather than under section 120.52(8)(d), as cited in the Rule Challenge Petition, and has addressed this ground in the Conclusions of Law section dealing with that alleged invalidity ground. section 471.005(7); the PP exam does not reliably determine if an examinee is minimally competent, due to an incorrect reference point; and the PP exam does not reliably distinguish between individuals whose practice of engineering would protect the public health and safety and those whose practice of engineering would constitute a danger to public health and safety. Alleged Invalidity of Challenged Rule under Section 120.52(8)(f) In paragraph 20 of the Rule Challenge Petition, Petitioner alleges that the Challenged Rule is invalid pursuant to section 120.52(8)(f) because it imposes regulatory costs on the regulated person, county, or city, which could be reduced by the adoption of less costly alternatives that substantially accomplish the statutory objectives. To this point, Petitioner proposes a state- conducted investigation of an applicant as a substitute for the PP exam, and contends that "there's a premium associated with a national exam. It's also clear that the regulatory cost imposed on me and all future examinees could be substantially reduced if the Board conducted their exam as an investigation and did not incorporate the NCEES exam." However, as more fully discussed below, this challenge ground is time- barred by section 120.541(1)(g), and, therefore, is not a legally-cognizable basis for invalidating the Challenged Rule in this proceeding. The Parties' Stipulated Facts Petitioner is an Engineer Extern, Texas EIT 56990. Pursuant to section 471.015(1), the Florida Engineers Management Corporation "shall issue a license to any applicant who the Board certifies is qualified to practice engineering and who has passed the [FE] exam and the [PP] exam." Both the FE exam and the PP exam are created by NCEES, pursuant to section 455.217(1)(d), which states, in pertinent part: "a board . . . may approve by rule the use of any national examination which the [Department of Business and Professional Regulation] has certified as meeting the requirements of national examinations and generally accepted testing standards pursuant to department rules." Pursuant to section 471.015(1), Petitioner passed the FE exam on September 3, 2016. Passing the FE is a prerequisite to take the PP exam. Petitioner registered to take the NCEES computer-based PP exam for environmental engineering on April 22, 2020. He paid the $350 exam fee and additional monies for test preparation material, and spent at least 100 hours preparing for the exam. He was notified, on April 6, 2020, by Pearson Vue, the test center company, that the exam was cancelled due to the Covid-19 pandemic. On April 7, 2020, Petitioner registered to take the exam on July 15, 2020, which was the earliest available date for taking the exam in his local area. On April 9, 2020, Petitioner canceled his registration for the July 15, 2020, exam and decided to apply for licensure as a P.E. without passing the PP exam. Pursuant to section 471.015(2)(a)1., on May 18, 2020, Petitioner filed his application for licensure with the Board. The application provides information stating that Petitioner meets the requirements of section 471.013(1)(a)1., and has at least four years of active engineering experience of a character indicating competence to be in responsible charge. In compliance with the education requirements of section 471.013(1)(a)1., Petitioner earned a Bachelor of Science degree in Chemical Engineering from the University of Florida, which is accredited by ABET. Petitioner provided an official transcript to the Board for verification purposes. In compliance with rule 61G15-20.002, Petitioner states in his application that he has at least four years of acceptable engineering practice, consisting of one year of experience equivalent through his Master of Engineering degree from the University of Texas at Austin, an ABET- accredited institution; and over three years of professional engineering work verified by licensed engineers. According to rule 61G15-20.002, an applicant must list three current personal references who are professional engineers. Thirteen licensed engineers submitted documentation to the Board regarding Petitioner's type of qualifying experience, level of engineering competency, and professional integrity. All references circled "yes" to the question "would you employ the applicant in a position of trust?" Two additional references in the engineering industry also provided their opinion on Petitioner's integrity and competence. On June 1, 2020, Petitioner received notification from the Board that his Florida 0901 1031-P.E. Endorsement application was incomplete. Petitioner addressed all items listed in a timely fashion to participate in the June 10, 2020, Board meeting. The Board stated during an informal hearing on June 10, 2020, that Petitioner would not be granted P.E. licensure due to not having passed the PP Exam. Petitioner is substantially affected by the Challenged Rule because it disqualifies him from becoming a licensed engineer in Florida without passing the PP exam. Findings of Fact Based on Evidence Presented at the Final Hearing A. Findings Regarding the Evidence Presented in Petitioner's Case Testimony of Witnesses Hoot and Grace David Hoot and Nigel Grace, both of whom are licensed professional engineers, testified regarding Petitioner's abilities and skills as a practicing engineer. Neither Hoot nor Grace was qualified as an expert in the field of psychometrics or related topics. Therefore, any opinions regarding these subject matters to which they testified at the hearing have been treated as personal opinions, rather than expert opinions. Hoot characterized Petitioner as a good, diligent young engineer, and described Petitioner's role in various projects on which they worked together. Based on his work with Petitioner, it was Hoot's personal opinion that Petitioner possesses the integrity and competence to work as a licensed engineer who would serve the public health and safety. Hoot testified that when he took the PP exam approximately 38 years ago, it was a free response exam. He stated that he was "not exactly sure" that a multiple-choice exam captures an examinee's ability to apply reason and judgment, but he acknowledged that he does not know how the PP exam currently is developed. He offered his personal opinion that it was possible for a competent engineer to fail the PP exam. Hoot also offered his personal view that engineering licensure constitutes a standard of care; however, he did not think anything provides a guarantee of competence. It was his personal view that experience tended to make one more competent. He also offered his personal view that as an engineer gains more experience and becomes more specialized, it is understandable that he or she would not score as well as an engineer who is gearing up toward the four-year experience goal which enables them to apply for licensure as a P.E. As Hoot put it, "life happens. You have children involved. You move jobs. I think you . . . maybe have less time to study. . . you get farther away from the study habits of . . . learning to be able to take tests." Grace, who is employed as a P.E. with Brown and Caldwell ("B&C"), a large U.S. engineering firm, testified regarding Petitioner's work experience while he was employed by B&C. Petitioner's experience included working on drinking water projects, utilities, upgrading process equipment, site evaluations, bench scale testing, and other projects. Based on Grace's work with Petitioner, it was his personal opinion that Petitioner possesses the integrity and competence to work as a licensed engineer who would serve the public health and safety. Grace took the PP exam approximately 28 years ago, and at the time, a major portion of the exam consisted of long-form written exam questions that provided the opportunity for examinees to provide free response answers and earn partial credit for partially-correct answers. He testified that the exam also had a multiple-choice component. Grace testified that, "based on instinct," he knows well-designed multiple-choice questions can provide the same insight into an examinee's decision-making judgment as long-form questions. Grace's personal view is that passing the PP exam does not guarantee competence, and it is possible for an engineer to be competent in some engineering disciplines but not others. Grace also agreed that it was logical that an examinee with approximately four years' experience would perform better on the PP exam because he or she would have better-honed test-taking skills and be fresher in some areas tested on the exam. By contrast, engineers who have practiced longer have more experience, but often have become specialized and further removed from the test-taking environment. Thus, passing a broad-spectrum exam becomes a bigger hurdle for engineers who have practiced longer. Petitioner's Testimony Petitioner testified on his own behalf at the final hearing. As discussed above, Petitioner holds a bachelor of science degree in chemical engineering from an ABET-accredited institution, and holds a master of engineering degree from an ABET-accredited institution. Although Petitioner, through his training as an engineer, is skilled at mathematics and statistics, he is not trained, and does not have any substantial experience, in the field of psychometrics.5 5 As discussed in greater detail below, psychometrics is a specialized field of study that concerns the theory and technique of objective psychological measurement of skills, knowledge, abilities, and educational achievement. Petitioner acknowledged that he does not have a degree in psychometrics and that he is not trained in, or knowledgeable about, preparing and administering high-stakes professional examinations. At the time he filed the Rule Challenge Petition, Petitioner had no personal knowledge of the work done by psychometricians; did not know what a "cut score" was; and was not familiar with the Modified Angoff Method, item response theory, the specifics of converting raw scores to scaled scores, or other any psychometric tools and methods employed to prepare and score the PP exam. Petitioner acquired some rudimentary knowledge in a few of these areas in preparing for the final hearing in this proceeding.6 Section 90.701(2), Florida Statutes, prohibits a lay witness from testifying as to an opinion regarding a matter involving specialized knowledge.7 Accordingly, it is determined that Petitioner is not competent to provide an expert opinion regarding psychometrics and related areas, such as item response theory. He was not qualified, tendered, or accepted at the final hearing as an expert in psychometrics or any related areas. Because Petitioner was not qualified to testify as an expert witness at the final hearing, Petitioner's testimony regarding psychometrics, item response theory, scaled scores, the Modified Angoff Method, high-stakes professional testing, and all other specialized subject matters, consisted of opinion testimony by a lay witness. The only instances in which a lay witness 6 Petitioner does not have any special knowledge, formal training, education, or experience in the specialized field of psychometrics. His knowledge about these areas was acquired by reading and study in preparation for the final hearing. Because Petitioner lacks special knowledge, experience, training, and education in psychometrics, he is not qualified to testify as an expert in psychometrics or related topics, such as item response theory. See Chavez v. State, 12 So. 3d 199, 205 (Fla 2009)(in determining whether a witness is qualified to render an opinion as an expert in a specialized field, the court must determine whether the witness is adequately qualified to render an opinion based on special knowledge, experience, training, or education). 7 Chesser v. State, 30 So. 3d 625 (Fla. 1st DCA 2010)(it is error for a court to accept opinion testimony of a lay witness in a specialized subject matter area). may provide opinion testimony are when the lay witness's opinion is based on firsthand knowledge through personal perception.8 As the undersigned explained during the final hearing, rather than excluding Petitioner's lay opinion testimony, she would consider these pertinent evidentiary principles in determining the weight to be afforded Petitioner's lay opinion testimony in this proceeding. Applying these evidentiary principles, it is determined that Petitioner's testimony regarding psychometrics and related topics, such as item response theory, classical response theory, and high-stakes test reliability and validity, concerned specialized subject matters not within the realm of common knowledge or based on Petitioner's personal perception. Rather, such specialized subject matters required expert witness testimony, pursuant to section 90.702, and as addressed above, Petitioner was not shown to be an expert in any of these specialized subject matters. Because Petitioner's testimony constituted the type of opinion testimony that is not permissible by a lay witness, pursuant to section 90.701, such testimony is not afforded weight in this proceeding. Petitioner testified that "minimal competence," which is the standard measured on the PP Exam, equates to "competence" as defined in the dictionary—that is, the "sufficient ability for a specific need or requirement." However, this position ignores that, for purposes of the Challenged Rule, "minimal competence" is a term of art specifically used, in the psychometric measurement context, to describe the level of competence that corresponds to a passing score on the PP exam. As more fully discussed below, the PP exam is developed and scored using psychometric tools and methods. 8 Nat'l Commc'ns. Indus., Inc. v. Tarlini, 367 So. 2d 670, 671 (Fla. 1st DCA 1979)(lay witness testimony regarding a specialized subject matter was not admissible into evidence because the testimony was not regarding a subject matter about which the witness could testify based on common knowledge or his personal perception.) Petitioner contends that engineering experience is, by itself, a reliable measure of competence, so it is unnecessary to also pass the PP exam. Petitioner testified "the application process and [PP] exam have two entirely different methods to identify the same result: whether an engineer in training is competent enough to become a licensed engineer It is not logical for two checks of competence to come up with different results. There should be consistency." In support of this position, Petitioner relies on section 471.015(2)(a), which requires "at least 4 years of active engineering experience of a character indicating competence to be in 'responsible charge' of engineering." § 471.015(2)(a), Fla. Stat. "Responsible charge" is defined in rule 61G15-18.011(1) as the degree of control an engineer is required to maintain over engineering decisions made personally or by others over which the engineer exercises supervisory direction and control authority. An engineer in responsible charge is the "engineer of record," as defined in rule 61G15-30.002(1). Rule 61G15-30.002(1) defines "engineer of record" as a Florida professional engineer who is in responsible charge. Thus, an engineer who is qualified, for purposes of being in responsible charge pursuant to section 471.015(2)(a) must, in addition to having the minimum statutory experience, be a licensed P.E. This means that he or she necessarily must have passed the PP examination. These statutory and rule provisions collectively reinforce the point that for an engineer to demonstrate competence for purposes of holding himself or herself out as an "engineer," as defined in section 471.005(7), he or she must satisfy all three requirements of section 471.015(2)(a)—i.e., education, experience, and passing the licensing exam.9 9 The requirement to meet these three requirements, including the PP exam, is codified in section 471.051(2)(a). Eliminating the exam requirement and relying strictly on education and/or experience for licensure would require the Florida Legislature to amend this statute Petitioner echoed the testimony of Hoot and Grace that licensure is not a guarantee of competence, and that passing the PP exam does not guarantee minimal competence. To this point, he testified that he does not believe that the PP exam adequately and reliably measures an applicant's ability to practice engineering, and that experience is a better indicator of competence than passing the exam. By way of example, Petitioner described his own experience10—which he characterized as "directly matching" the activities in which a licensed engineer engages—and compared that experience to measuring competence by an exam, which Petitioner characterized as "attempt[ing] to indirectly measure my ability as an engineer." Based on his personal experience, Petitioner contends that experience better demonstrates competence to be licensed as a P.E.; that passing the PP exam does not indicate minimal competence to practice engineering; and that failing the PP exam does not mean that the examinee is not minimally competent. He further testified that examinees who fail the PP exam likely are minimally competent, since the engineering jobs they hold when applying for licensure likely would require that they be minimally competent in order to have been hired.11 to eliminate the exam requirement. The undersigned is not authorized by statute or the Florida Constitution to eliminate the PP exam requirement for licensure under chapter 471. 10 Petitioner's experience, set forth in his P.E. licensure application, was verified by his supervising engineers. 11 Petitioner appears to conflate being determined not "minimally competent" for purposes of passing the PP exam, with "incompetence," which is defined in Florida Administrative Code Rule 61G15-19.001(5) as the "physical or mental incapacity or inability of a professional engineer to perform the duties normally required of the professional engineer." Part of this confusion may be due, in part, to Respondent's response to one of Petitioner's interrogatories asking for a definition of "minimal competence." Rather than directly answering the interrogatory, Respondent referred Petitioner to the definition of "incompetence" for purposes of imposing discipline under Respondent's disciplinary rules—thus causing Petitioner to understandably assume that failing to demonstrate minimal competence through passing the PP exam equates to "incompetence," as defined in rule 61G15-19.001(5). However, the fact that Petitioner has not demonstrated "minimal competence" on the PP exam does not mean that he is incompetent; it simply means that he has not yet passed the PP exam for licensure as a P.E. in Florida. To this point, if failing to demonstrate "minimal competence" by passing the PP exam equated to being incompetence, every person who performs engineering work in To further illustrate this point, Petitioner noted that the data regarding passage rate of the PP exam shows that examinees having zero years of experience are almost twice as likely to pass the PP exam as examinees having 11 or more years of experience. However, as Hood and Grace explained, and as further discussed below, engineers having more than four to five years of experience begin to specialize in narrower fields and "life happens," in that personal and professional circumstances render it more difficult to prepare for and take a high-stakes test. Petitioner also disputed the accuracy of the PP exam preparation and scoring process. In particular, he took issue with the "model law engineer" standard to which the exam is designed. As discussed more extensively below, this standard equates to the competence level of an engineer having four years of engineering experience and who is capable of practicing engineering in a manner that protects the public health and safety. In particular, Petitioner contends that designing the PP exam to the "model law engineer" standard is unfair to anyone taking the exam that does not have exactly four of years of engineering experience. Notably, however, section 471.015(1), which is the statute implemented by the Challenged Rule, establishes four years as the engineering experience required for licensure as a P.E. Thus, the "model law engineer" standard is rationally related to the statutory minimum experience level for purposes of demonstrating minimum competency to be licensed. Petitioner also contends that the subject matters tested on the PP exam are unfairly broad, so that engineers who specialize in a particular area within an engineering discipline—such as specializing in water-related areas in environmental engineering—are disadvantaged by being required to take Florida but has not passed the PP exam would be "incompetent," and, thus, potentially subject to disciplinary action. an exam that covers a broad range of areas beyond his or her area of specialty. Petitioner further contends that it is irrational to test an examinee on particular areas that are irrelevant to his or her work and/or desired career path. However, the PP exam for a particular discipline is specifically designed to ensure that a licensed P.E. is competent to practice over a range of specific areas encompassed within that particular discipline. This is because once a person becomes a licensed P.E., he or she may practice engineering within any discipline or specific area within that particular discipline, subject to the professional and ethical requirements to limit practice to the disciplines and areas in which the engineer is actually competent. Thus, the breadth of the PP exam is designed to help ensure minimal competence to practice engineering in a manner that protects the public health and safety. Petitioner also contends that because the PP exam for some engineering disciplines tests a broader range of areas than the PP exam may test for other disciplines, the exam inconsistently measures minimal competency across the range of engineering disciplines. However, as discussed in detail below, the subject matters tested on the PP exam for a given discipline are chosen by subject-matter experts who are licensed engineers practicing in that particular engineering discipline, and are deemed, by those subject matter experts, to be most important to test for purposes of measuring competency in that discipline. Thus, while the number of discrete subject matters tested on the PP exams may differ across the various engineering disciplines, this difference is, factually and logically, a function of expert consensus regarding which subject matters need to tested to demonstrate minimal competence. Petitioner also contends that the Challenged Rule is vague because it does not specifically identify the disciplines, and the areas within each discipline, that are tested on the PP exam. Respondent has contracted with NCEES to be responsible for preparing, administering, and scoring the PP exams, pursuant to section 455.217(1)(d). NCEES conducts a methodical process, discussed in detail below, to determine the specific disciplines for which to develop a PP exam and the areas to be covered on the PP exam for a discipline. Exam specifications are then developed by subject matter experts within that discipline, and are published by NCEES. These specifications inform prospective examinees regarding the particular areas that will be tested on the PP exam for the discipline, and the number of questions for each specific area that will appear on the exam. Thus, prospective examinees are not left to wonder or guess about which disciplines will be tested; the areas within each discipline that will be tested; or the relative weight that will be assigned to each area tested.12 Although the Challenged Rule does not identify the specific disciplines tested on the PP exam, rule 61G15-21.001(1)(b)—which actually adopts the PP exam as an engineering licensure exam in Florida—states that the PP exam "is given by discipline." Therefore, even if section 455.217(1)(d) required the specific areas of competency to be identified by rule, such areas would have been identified in rule 61G15-21.001(1), rather than in the Challenged Rule.13 12 See Cole Vision v. Dep't of Bus. And Prof'l Reg., 688 So. 2d 404, 410 (Fla. 1st DCA 1997)(a rule is impermissibly vague if it is drafted in terms so vague that men of common intelligence must necessarily guess at its meaning or application. 13 Neither sections 455.217(1)(d) nor 471.015(1) specifically authorize or require Respondent to adopt rules identifying the general areas of competency tested on the PP exam. By contrast, exams developed by the agency pursuant to section 455.217(1)(b) must identify, by rule, the general areas of competency to be tested. Had the Legislature intended for exams authorized under section 455.217(1)(d)—of which the PP exam is an example—to adhere to the same requirement, the statute would have so stated. See Pro-Art Dental Lab, Inc. v. V- Strategic Grp., LLC, 986 So. 2d 1244, 1258 (Fla. 2008)(the specific mention of one thing in a statute implies the exclusion of another). Furthermore, section 120.54(1)(g) expressly requires a rule to address only one subject. Thus, if the Challenged Rule also addressed the areas of competency to be tested on the PP exam, it would violate section 120.54(1)(g). Petitioner also asserted, at the final hearing, that Respondent did not certify the PP exam as meeting the requirements of national examinations and generally accepted testing standards pursuant to department rules, as required by section 455.217(1)(d). However, Petitioner did not raise this alleged invalidity basis in the Rule Challenge Petition, so he is foreclosed from raising and litigating it at the final hearing. See § 120.56(1)(b), Fla. Stat. Petitioner also testified that, in transitioning from paper-and-pencil PP exams to computer-based exams, NCEES is relying on two different theories—classical test theory and item response theory—and that this reliance does not comport with generally accepted testing standards. However, as discussed above, Petitioner was not qualified as an expert in the specialized area of high-stakes examination preparation and scoring; thus, his testimony constitutes lay opinion regarding this specialized subject matter. He did not present any competent substantial evidence to support his contention that the PP exam does not meet generally accepted testing standards.14 Petitioner also testified that item response theory, which is a psychometric tool used in developing and scoring the PP exam, is an invalid means of determining the competence of an engineer. To this point, Petitioner testified that the "model law engineer" is not a real person, but is instead an imaginary person created by subject matter experts to define what a minimally competent engineer should know. Thus, according to Petitioner, 14 Because Petitioner was not qualified, tendered, or accepted as an expert in these specialized subject matters, and because his testimony on these matters consists of inadmissible lay opinion testimony, this testimony has not been afforded weight. §§ 90.701 and 90.702, Fla. Stat. the model law engineer standard is the wrong reference point for determining minimal competency to practice engineering.15 Petitioner also testified that the PP exam does not accurately measure ability, which is a latent trait for which an arbitrary measurement scale must be created. He testified that the model law engineer standard is the midpoint of this scale, and that the purpose of the scale is to determine whether examinees fall above or below that midpoint.16 He further contended that the PP exam does not accurately measure ability, because performance on the exam may be influenced by extraneous variables, such as test anxiety. Petitioner also testified regarding item response theory, which, as previously noted, is a psychometric tool used in developing and scoring high- stakes exams—a subject about which Petitioner had no training in, or knowledge about, until he prepared for the final hearing in this proceeding. Specifically, Petitioner testified that the item characteristic curve is the basic building block of item response theory, and that there are two technical properties of an item characteristic curve: difficulty of the item, and the ability of the item to discriminate between examinees' abilities. Petitioner testified that another basic principle of item response theory is that the examinee's ability is a variable with respect to the items used to determine it. According to Petitioner, this principle rests on two 15 Petitioner's testimony on this point was based on excerpts from a book titled Item Response Theory and a book titled The End of Average. The Item Response Theory book is a treatise on psychometrics, a highly specialized field about which Petitioner was not qualified to testify as an expert, and which is not susceptible to lay witness opinion testimony. Thus, Petitioner's testimony on these points is not assigned weight. See §§ 90.701 and 90.702, Fla. Stat. Additionally, excerpts from The End of Average were determined irrelevant, so were not admitted into evidence. 16 As support for this testimony, Petitioner selectively cited and quoted the deposition testimony of Timothy Miller, Respondent's expert on the development and scoring of NCEES's psychometric-based PP exams. The specific context of Miller's deposition testimony was that when an exam item is overexposed, it is subject to drift, which means that the percentage of correct answers for the item increases to the point that the item no longer is a good discriminator. As further discussed below, Petitioner's testimony on this point was directly and persuasively countered by Miller's expert testimony regarding scaled scores and setting the passing score for the PP exam. conditions: that all items measure the same underlying latent trait, and the values of all item parameters are in a common metric. According to Petitioner, this principle reflects that the item characteristic curve spans the entire ability scale; thus, the practical implication is that a test located anywhere along the ability scale can be used to estimate an examinee's ability, such that an examinee could take a test that is easy or hard, and on average, would score at the same estimated ability level. Petitioner testified that this stands in contrast to classical test theory, which he contends is a better discriminator of examinee ability.17 Petitioner also testified that the psychometric methods used to develop and score the computer-based PP exams are flawed because "difficulty" is subjective and entirely dependent on the individuals developing the PP exam. Thus, according to Petitioner, in scoring a computer-based multiple-choice PP exam, it is impossible to know whether a particular examinee got the answer right due to a reasonable approach in answering the question, or by guessing. Petitioner contends that for this reason, multiple-choice test questions developed using item response theory are not good discriminators of examinees' ability; thus, even if an examinee does not correctly answer enough questions to pass the exam, that does not mean that the examinee is not knowledgeable in that area.18 Petitioner further testified that because difficulty is a subjective parameter, different forms of the PP exam inherently have different levels of difficulty. Thus, according to Petitioner, it is a matter of luck whether an examinee takes a more difficult form or an easier form of the exam. Further to this point, Petitioner testified that because an examinee does not take multiple forms of the exam, but instead takes only one form, the determination of the examinee's ability is solely dependent on a subjective 17 Refer to note 15, supra. 18 Refer to note 15, supra. parameter—i.e., the difficulty of the test questions as determined by subject matter experts. Petitioner contends that, as a result, the PP exam does not accurately measure an examinee's ability, and, therefore, is not a valid exam.19 Petitioner also testified that because statistical indices of reliability and validity are not attributes of an exam, a researcher may select what seems to be an appropriate test for his or her purposes, when, in fact, the selected test does not have any level of reliability or validity. Thus, Petitioner testified, reliability and validity are values that reside in test scoring, not in the test itself. Petitioner testified that validation, in statistics, is the process of accumulating evidence that supports the appropriateness of the inferences that are made of student responses for assessment uses. He testified that validity refers to the degree to which the evidence indicates these interpretations are correct and the manner in which the interpretations are used is appropriate.20 Petitioner testified regarding three types of validity evidence: content, construct, and criterion evidence.21 Specifically, Petitioner testified that content evidence refers to the extent to which an examinee's responses to a given assessment reflect the examinee's knowledge of the content being tested; thus, to the extent an exam inadvertently measures a parameter that is not related to the examinee's knowledge of the content being tested, it is invalid. 19 Refer to note 15, supra. 20 Petitioner's testimony relied on, or was paraphrased from, a document titled The Scoring Rubric Development. Again, because this topic and document address a matter within the specialized fields of psychometrics, high-stakes testing, and test-scoring statistics, which are areas in which Petitioner was not qualified as an expert, and which are not susceptible to lay opinion testimony, pursuant to sections 90.701 and 90.702, Petitioner's testimony relying on this document, including his testimony regarding content, construct, and criterion-related evidence, is not afforded weight. 21 Refer to note 15, supra. This determination regarding the weight afforded Petitioner's testimony applies to paragraphs 94 through 100 herein. According to Petitioner, the content-related evidence for the PP exam for each discipline is inconsistent, so that the PP exam for a given discipline does not accurately measure minimal competence for that discipline. Petitioner also testified that the weighting of different topics on the PP exam necessarily creates an advantage for engineers who work in areas more heavily weighted on the exam, while creating a disadvantage for engineers who work in areas that are less heavily weighted on the exam. Petitioner also testified that to accurately determine minimal competence in all engineers, the model law engineer standard should be keyed to, and the content tested on the exam should be directed toward all engineers, including those having more than four years of experience. Petitioner also testified that construct-related evidence consists of external benchmarks, such as results and explanations, of internal evidence of psychological processes, such as reasoning. Petitioner testified that because multiple-choice exams do not provide evidence of an individual's reasoning process, they do not generate construct-related evidence for purposes of determining exam validity. Petitioner testified that free response paper-and pencil-exams provide construct-related evidence, so are better indicators of an examinee's knowledge. Petitioner also testified regarding criterion-related evidence, which relates to the extent to which the results of an assessment, such as the PP exam, correlate with a current or future event. By way of illustration, Petitioner testified that criterion-related evidence considers the extent to which a student's performance may be generalized to other relevant areas. Petitioner testified that an examinee's performance on the PP exam is not generalizable to other relevant activities, so it is impossible to determine whether the exam actually corresponds to minimal competence in the workplace. In sum, Petitioner testified that the PP exam does not meet content, construct, or criterion-based evidence for purposes of determining whether it is a valid exam. Thus, Petitioner reasons, it is logical to conclude that because the PP exam is not a good discriminator between minimally competent and incompetent engineers, it does not reliably and adequately measure competence. Petitioner also testified that because passing the PP exam is only one component of licensure, it fails to meet criterion-based validity, in that the exam, by itself, does not certify a passing examinee to practice as a P.E. As Petitioner put it, "you're just passing the exam as part of the requirement for licensure." Petitioner reasoned that if passing the PP exam corresponds to minimal competence, the experience and education requirements of section 471.015(2)(a) are redundant. Also to this point, Petitioner testified that the preapproval process to take the PP exam is directly related to an examinee's actual work experience as an engineer, while taking the exam merely entails answering questions about engineering work. Thus, Petitioner contends, a competent engineer, as shown through Respondent's preapproval process, may nonetheless fail the exam. Petitioner asserts that this further shows that the exam does not accurately measure minimal competence. Petitioner also testified that, in his view, delaying licensure of potentially competent engineers due to postponing the exam due to the Covid-19 pandemic does not serve the interest of public health and safety. To that point, he testified that the inability to obtain a variance, which would relieve examinees from having to take and pass the exam under such circumstances, renders the Challenged Rule arbitrary.22 Petitioner also contended that passing the PP exam should not be required, because other engineering professional associations—specifically, the European Federation of National Engineering Associations ("FEANI"), which represents engineers in European countries—allow licensure through 22 Petitioner's point regarding inability to obtain a variance or waiver is addressed in the Conclusions of Law. education and experience requirement, without requiring a professional exam to be taken and passed. However, because section 471.015, which governs the licensure of engineers in the state of Florida, requires a professional licensure examination to be taken and passed as part of the P.E. licensure requirements, Petitioner's testimony and argument regarding FEANI's practices and requirements take issue with the statute, rather than the Challenged Rule, and, thus are irrelevant. Petitioner also contends that the examination fee for the PP exam is arbitrarily set, rendering the Challenged Rule arbitrary. However, as discussed above, the Challenged Rule only addresses determining the passing grade for the PP exam using psychometric methods. The Challenged Rule has nothing to do with establishing or setting an examination fee. Thus, this challenge ground has no basis in fact or law.23 C. Findings Regarding the Evidence Presented in Respondent's Case Respondent's Expert Witnesses Respondent presented the testimony of Timothy Miller, P.E., who serves as Director of Examination Services for NCEES. Miller has held this position for approximately 15 years. His job-related activities and responsibilities include directing exam development, publication, scoring, and fulfillment of the licensing exams for engineers and surveyors; coordinating exam development committees consisting of over 800 volunteer subject matter experts who work on developing each NCEES exam; overseeing the exam development process and providing advice and guidance regarding engineering exam development, administration, production, scoring, analysis, and reporting; serving as a testing process consultant to exam development committees; and other exam-development and administration-related matters. 23 Additionally, this challenge ground was not raised in the Rule Challenge Petition, so is not at issue in this proceeding. See § 120.56(1)(b), Fla. Stat. Before Miller was promoted to his current position, he served as an examination development engineer for NCEES. In that position, Miller was responsible for planning and coordinating engineering exam development, production, administration, scoring analysis, and reporting for certain assigned examinations; serving as a testing consultant working with engineering exam development committee chairs regarding quality and number of exam development volunteers; and overseeing development and administration of the licensing exams in the specific fields of environmental controls systems, metallurgical engineering, and mechanical engineering. Through his experience in these positions with NCEES, Miller is an expert in professional examination development and scoring, particularly with respect to the development and scoring of the NCEES FE and PP examinations. Before being employed with NCEES, Miller practiced civil and structural engineering with several private-sector engineering firms. He has been a professional engineer since 1984, and is licensed as a P.E. in South Carolina, North Carolina, Maryland, Delaware, Pennsylvania, and New Jersey. Respondent also presented the testimony of Dr. Michelle Rodenberry, P.E., an associate dean and associate professor at the Florida A&M University–Florida State University College of Engineering. Her engineering expertise is in the field of structural engineering—specifically, bridge engineering. Rodenberry was appointed to the Board in 2012, and she served as a Board member until 2018. She is now an emeritus Board member. While on the Board, she served as chair of the education committee, and was involved in reviewing applications for licensure as a P.E. in Florida. Development, Scoring, and Validation of the PP Exam The NCEES engineering exams are national licensing exams that are recognized by every engineering licensing entity in each of the U.S. states, as well as by the engineering licensing entities in Washington, D.C.; Puerto Rico; the U.S. Virgin Islands; and the other U.S. territories and protectorates. There are approximately 26 different engineering disciplines, each of which is tested by a separate PP exam specific to that discipline. In the 1990s, NCEES decided to transition from subjectively-graded pencil-and-paper examinations to an objectively-graded computer-based multiple-choice exam format. Currently, approximately one-third of the PP exams, including environmental engineering, have been converted to a computer-based format, and all but one of the exams in the other disciplines is in the process of being converted. The reason NCEES is transitioning the PP exam from a pencil-and- paper format to a computer-based format consisting of multiple-choice questions is to help eliminate subjectivity in grading, so that the exam papers are consistently graded across groups of examinees. Additionally, a computer-based format consisting of objective multiple-choice questions allows the difficulty of the exam to be psychometrically evaluated for purposes of determining the passing score for a particular administration of the exam. To that point, because computer- based multiple-choice exams are objectively scored, exams offered at different times during the year are able to be compared, or equated, for purposes of setting the passing grade for a particular exam administration.24 Respondent entered into a contract with NCEES in 2009, pursuant to which NCEES provides the FE and PP exams for engineer licensure in Florida. In 2013, the contract was amended to allow NCEES to provide the exams by computer-based testing, using Pearson Vue as its exam 24 As Miller explained,"[i]f they were different on a difficulty level, the harder exam, the standard would actually be lowered so that it would be fair across administration so everybody was treated consistently. Or if my exam was less difficult, the standard would be raised. I would have to get more questions right." administering entity. The FE and PP exams are administered by Pearson Vue at its testing centers. NCEES develops model laws and rules that represent best practices with respect to state licensing of engineers. The aim of these model laws and rules is to achieve uniformity and consistency throughout the states and the U.S. territories and protectorates in the licensure of professional engineers. A significant benefit of such consistency and uniformity is the resulting "mobility" for licensed professional engineers—that is, the ability to more easily become licensed to practice engineering in multiple states. The NCEES model laws and rules establish the "model law engineer," which defines and constitutes the standard for minimal competence in a specific engineering discipline for purposes of being licensed as a P.E. in that discipline. The model laws and rules define the "model law engineer" as a person who holds a degree from an engineering educational program accredited by ABET, has four years of active engineering practice experience, and passes the FE and PP exams. The model law engineer standard equates to the competence level of an engineer having four years of engineering experience who is capable of practicing engineering in a manner that protects the public health and safety. This constitutes the minimum competence level that an applicant must demonstrate for purposes of being licensed as a P.E.25 in the 50 states and the U.S. territories and protectorates. Thus, the NCEES PP exam is constructed to test engineering ability keyed to the model law engineer standard. That is, the PP exam is designed to determine the ability level of an applicant for P.E. licensure for purposes of 25 Refer to note 12, supra. The term "minimal competence," as used in the Challenged Rule, is specifically keyed to the "model law engineer" standard for purposes of being licensed as a P.E. It is not meant to indicate or imply that an engineer who does not take or pass the PP exam is per se incompetent, such that he or she is not competent to engage in work constituting engineering, as defined in section 471.005(7). comparing that ability level to that of an engineer having four years' experience who is able to practice engineering in a manner that protects the public health and safety. Examinees having four years of engineering experience after graduation have the highest pass rates on the PP exams. Pass rates for examinees with more or less than four years of experience are lower, typically in proportion to the length of time before or after the four-year experience mark when they take the PP exam. Miller explained, credibly and persuasively, that the reason for the drop-off in PP exam performance after the four-year mark is that "life happens." Engineers gain more experience, and many become specialized in a relatively narrow niche, or move into managerial, non-technical positions. Additionally, because the PP exam does, in part, test subjects that one learns in college, the longer an examinee is out of college, the less subject matter recall in certain areas he or she may have. "Psychometrics" is the specialized field of study concerned with the theory and technique of psychological measurement. Specifically, psychometrics entails the objective measurement of skills and knowledge, abilities, and educational achievement. Among other specialized areas of practice, psychometricians focus on the construction and validation of assessment instruments, and theories, such as item response theory, that relate to psychological measurement. Psychometricians typically have graduate training and all possess specialized qualifications that enable them to engage in objective psychological measurement. PP exams are designed to determine minimal competence in a specific engineering discipline. "Minimal competence" is the minimal amount of knowledge required to practice in that particular engineering discipline in order to protect the public health and safety. For any specific engineering discipline for which it has been determined that a PP exam should be given,26 there is an approximately three-year due diligence period in which subject matter experts in that discipline work to determine the topics that should be tested on the exam. The PP exam for each specific discipline is developed by subject matter experts, who volunteer and meet on a monthly basis to develop, review, and evaluate the questions for the PP exam for that specific discipline.27 The process of determining which topics should be tested on a PP exam, termed the Professional Activities and Knowledge Study ("PAKS") process, is a standard practice used to determine the specific topics to be tested on a PP exam. As part of the PAKS process, a consulting psychometrician28 employed by Pearson Vue; 15 to 20 engineers who are licensed in another engineering discipline; and subject matter experts who may teach a particular engineering discipline for which the PP exam is being developed, work together to develop consensus regarding the specific topics that engineers having four years of experience practicing in that discipline need to know in order to safely practice engineering in a manner that protects the public health and safety.29 26 For a PP exam to be developed for a new engineering discipline, at least ten NCEES- member state engineering boards must request that such exam be developed, and at least one ABET-accredited program in that specific discipline must exist. 27 Over the years of development and administration of the PP exam, hundreds of licensed engineers have provided input regarding the topics that should be, and are, tested in each discipline and the relative weight given to each topic on the PP exam. 28 Pearson Vue's psychometricians who develop, score, and evaluate NCEES's exams have Ph.D. degrees in psychometrics or statistical analysis. 29 Subject matter experts selected to develop the PP exam questions are chosen based on consideration of the type of practice, such as governmental and private practice; gender; ethnicity; length of time of licensure as a P.E.; and geographic considerations. All subject matter experts must be licensed as a P.E. by a state engineering licensure board in order to The consulting psychometrician builds a questionnaire that lists the specific topics identified by the PAKS committee, and distributes an online survey to engineers who practice in the discipline for which the PP exam is being developed. The survey seeks input regarding the relative importance of each specific topic for purposes of testing to demonstrate minimal competence in the discipline. Based on the survey responses from engineers practicing in the discipline, exam specifications are developed. The exam specifications identify each specific topic to be tested on the PP exam, and the number or percentage of exam questions that will address each specific topic within that discipline. The exam specifications must be approved by an oversight committee. Once the exam specifications have been approved, the subject matter experts for that specific engineering discipline for which the PP exam is being developed prepare the PP exam questions—also termed "items"—and review and evaluate them for clarity, demographic neutrality, and other parameters, so that the items will reliably and validly test engineering ability. In computer-based multiple-choice PP exams, the questions are prepared such that for each question, there is only one correct answer and three other plausible, but incorrect, alternative choices. The individual exam questions are reviewed numerous times by the subject matter experts before they are moved into an exam question bank for use on the PP exam. Once the exam questions have been developed and banked for use on a PP exam, a standard-setting committee, consisting of ten to 15 licensed engineers having diverse backgrounds, experience, and demographic features, reviews the exam to determine the minimum passing score—or "cut score"—on the exam. The cut score equates to the ability level of an engineer serve in this capacity. As noted above, over the years of development and administration of the PP exam, hundreds of engineers have provided input in developing each PP exam. having four years of experience who is minimally competent to practice engineering at a level that protects the public health and safety. This method of using subject matter experts to examine the content of each exam question and predict how many minimally-qualified examinees would answer each question correctly is termed the "Modified Angoff Method." The standard-setting committee then takes the exam, and the psychometrician analyzes the data from the committee's exam sitting. Using this data and analysis, the standard-setting committee then reviews, and reaches consensus, regarding each question, for purposes of determining the proportion of minimally competent engineers who would answer that specific question correctly. Based on the information generated by this process, the psychometrician develops the "panel recommended passing score," with a statistical margin of error. The psychometrician presents this recommended passing score to a committee of five persons, consisting of two state licensing board members and three subject matter experts who observed the exam development process. Based on the psychometrician's recommendation, the committee makes the final decision regarding the minimum passing score for the exam. Each PP exam question is developed and evaluated using the process described above, and is placed in bank for use on a PP exam. The psychometrician uses item response theory to "calibrate"—i.e., determine the relative difficulty level of—each exam question.30 An exam question is not banked for use on future sittings of the exam unless it has had at least 200 responses on a previous exam, so that statistics for each item's performance can be generated for purposes of item calibration. 30 Item response theory is one of many psychometric methods, or tools, used to weight exam questions for purposes of creating different forms of exams having the same level of difficulty. Depending on the specific discipline, a question bank for a PP exam may consist of many thousands of questions.31 Using item response theory, the psychometrician converts the passing score to create a scale from -5 to +5, which will equate to the examinee's ability level as measured by the exam. Once the passing score for the PP exam has been determined, different PP exam "forms" are created for administration in different exam sittings. Exam "forms" are essentially different versions of the PP exam that consist of different individual questions of the same difficulty level, as determined using item response theory, for each specific topic on the exam. Thus, if a PP exam was administered, for example, in April and October, the different exam administrations would consist of different forms—meaning that the exam would consist of different questions, but the questions would be of the same difficulty level for a specific topic tested on the exam.32 Additionally, because exam item difficulty has been calibrated using item response theory, different forms of a PP exam can be given during the same exam administration at different locations.33 Importantly, because the difficulty of the exam items has been calibrated using item response theory, the different exam forms are statistically equivalent in difficulty. 31 The only items that will be used on the graded portion of the PP exam are questions that have known statistics such that they have been calibrated for difficulty. However, there may be other "pretest" questions on the exam strictly for purposes of gathering statistics regarding performance on the questions for potential inclusion in the exam item bank; these "pretest" exam questions are not graded for purposes of determining the examinee's score on the exam. 32 As Miller explained, for an administration of an exam at different locations at the same time, the form administered at a particular location consists of different questions than the form administered at another location; however, the exam forms are equivalent in terms of the number of questions addressing a particular topic and the difficulty of the items addressing that topic. 33 Using the "linear-on-the-fly" ("LOFT") method to generate different forms of the exam also helps ensures exam security, since persons sitting near each other during an administration of the exam will not have the same exam form. As a result of using these processes, including the Modified Angoff Method, and applying item response theory to calibrate the exam items for purposes of constructing different, but statistically equivalent, forms of the PP exam, examinees are not graded on how they perform against each other, but instead are graded against the cut score set for the exam. To ensure that different forms of the exam are statistically equivalent in difficulty, Pearson Vue uses the LOFT method,34 which employs an algorithm to ensure that, across all of the exam forms, all examinees get the same number of questions of the same level of difficulty on the same topics. The algorithm randomly generates, or assembles from banked exam questions, different exam forms based on the exam specifications (i.e., the specific topics tested and relative weight/number of exam questions for that topic) and the difficulty level of the exam questions, such that the different exam forms generated by LOFT are statistically equivalent to each other. Using item response theory to calibrate specific exam question difficulty based on the statistical probability of being answered correctly enables examinees taking different, but statistically equivalent, forms of the exam to be compared to the passing standard for purposes of determining whether they pass the exam. Thus, examinees are compared to an ability level—here, minimal competence—rather than to each other.35 This method ensures that all examinees take an exam of equivalent difficulty, which, in turn, helps ensure the fairness of the exam. 34 For the engineering disciplines having too small a group of examinees to employ item response theory or LOFT to generate different exam forms, each examinee takes the same exam instead of taking different forms of the exam, and the exam typically is offered only on one day, rather than multiple days, per year. 35 By way of example, Miller explained that if two examinees each answer five questions on the same topic on the exam, and one examinee answers four easier questions correctly and the other examinee answers two comparatively more difficult questions correctly, the examinee answering the two more difficult questions correctly may have a higher ability level on that particular topic, due to the comparative difficulty of the questions that examinee answered correctly. Once a PP exam is administered, Pearson Vue scores the examination and sends NCEES the information regarding whether each examinee has passed or failed the exam. Pearson Vue also provides each examinee the information regarding his or her performance on the exam compared to the minimum competence standard. The examinee's performance is expressed as a scaled score, for each specific topic tested on the exam, and for the entire exam. Specifically, using psychometric statistical methods, the ability level of the examinee is expressed as "theta," and is placed on a scale of 0 to 15 for each of the specific topics tested. The examinee's overall theta across all specific topics tested is then compared to the "minimal competence" passing standard, which is also expressed as a scaled score using the same 0 to 15 scale. After an exam is taken and scored, the consulting psychometrician analyzes this data, called "response data," for each exam question, for each examinee, to calibrate the items for purposes of determining whether a particular question performs well in discriminating ability level of the examinees. The psychometrician may recommend that an exam question be "retired" because it is not performing as a good discriminator of ability level. Examples are where an exam question is too difficult or too easy, such that it does not discriminate well in determining ability level; where an item takes too long to answer or is ambiguous; where an item has been "overexposed" by having become publicized such that future examinees have access to the question and scores on the question become high; or where an exam contains "bad pair" items, such that the answer to one item may suggest, or lead to, the answer on another similar item. Having a psychometrician involved in tracking and analyzing exam data enables such circumstances and situations—which may influence the scores on a test item for reasons not related to the examinee's ability—to be identified and corrected. Returning to a free response, paper-and-pencil exam format for the P.E. licensing exam would provide a far less objective, fair, and accurate measure for determining minimal competence for purposes of being licensed to practice engineering. It also would negatively affect the ability of licensed engineers to become licensed in other states. Due to the use of psychometric methods in developing and scoring, the PP exams are very reliable across multiple administrations of the exam— to the point that all of NCEES's psychometric-based PP exams score upwards of .9 on a scale of 1.0.36 Psychometric methods, including item response theory, are used in developing, administering, and scoring many different types of high-stakes professional and academic examinations, including medical school admissions examinations, and nursing, medical examiner, internal auditor, and architecture licensure examinations. NCEES audits approximately one-third of the exams administered by Pearson Vue on an annual basis, to independently evaluate the accuracy of the psychometric services provided by Pearson Vue, and to ensure that the exams have been created pursuant to NCEES's guidelines, procedures, and requirements. NCEES also retains independent psychometricians to review Pearson Vue's exam-related reports and analyses, to ensure that Pearson Vue is following standard psychometric rules of good practice. In sum, the use of objective psychometric methods, including the methods discussed above, to develop, score, and evaluate the PP exam ensures that minimal competence, for purposes of licensure as a P.E., is accurately measured by the exam. Role of the PP Exam in Licensure of PEs in Florida As discussed above, to be licensed as a P.E. in Florida, an applicant must have a college degree from an ABET-accredited institution, four years of 36 Test reliability refers to the degree of consistency with which a test measures a particular subject matter across different administrations of the test. A test has a high reliability score if it consistently produces similar results under consistent conditions. A 1.0 reliability score reflects perfect consistency in results across different administrations of a test. An acceptable reliability score target for high-stakes tests is .7 or higher. active experience in engineering practice, and have passed the FE and PP exams. Thus, the P.E. exam is a vital component of determining that an engineer licensed as a P.E. to practice in Florida is able to practice at a competence level that protects the public health and safety. Unlike the education and experience requirements for licensure— both of which may entail a great deal of variability in quality and breadth across applicants—the PP exam constitutes an objective, consistent tool37 to measure an applicant's level of competence for purposes of determining whether the applicant possesses the minimal competence needed to practice engineering in a manner that protects the public health and safety. As such, the PP exam constitutes a uniform measure of minimal competency for purposes of licensure as a P.E. in Florida. As discussed above, this does not mean that a person who engages in engineering work but has not passed the PP exam is incompetent; it merely means that he or she has not demonstrated minimal competency on this required objective measure of competency for licensure purposes. As discussed above, the PP exam is specifically designed to ensure that a licensed P.E. is competent to practice over a range of specific areas encompassed within a particular discipline. This is because a licensed P.E. is authorized to practice engineering within any discipline or area, subject to professional and ethical standards. The breadth of the PP exam thus helps ensure minimal competence to practice engineering in a manner that protects the public health and safety. 37 This consistency and uniformity is the direct result of the psychometrically-based exam development, scoring, calibration, and validation methods discussed above. The purpose of P.E. licensure is to inform and protect the public, which is entitled to rely on such licensure as indicating that the licensee is competent to practice engineering.38 Administration of the PP Exam During the Covid-19 Pandemic As discussed above, Petitioner has alleged that the Challenged Rule is arbitrary on the basis that it does not address contingencies for offering the exam if unforeseen circumstances prevent regular administration of the PP exam. Specifically, Petitioner points to the fact that Pearson Vue cancelled the April 2020 PP exam administration due to the Covid-19 pandemic. Pearson Vue has rescheduled the PP exams for various times and at various locations around the country in an effort to make the PP exam available for prospective examinees during the pandemic.39 Pearson Vue also is taking substantial steps to protect persons who have applied to take the PP exams during the Covid-19 pandemic. To that point, Pearson Vue has retrofitted its testing centers to help ensure the safety of the examinees as they sit for the PP exam. Specifically, the number of examinees who will be in a testing room has been reduced; masks are required to be worn by examinees and proctors; testing stations are cleaned between each use; some additional test center locations have been added; and some states have relaxed rules to allow examinees to sit for the exam in states other than the one for which they are applying for licensure. At present, the exams are not able to be offered over the internet so that examinees are able to take the exam at a remote location. A substantial reason for this is lack of exam security, which is necessary to protect and 38 As noted above, a person does not have to be licensed as a P.E. to engage in engineering work in Florida. However, if a person wishes to hold himself or herself out to the public as a P.E., then that person must satisfy the requirement to pass the PP exam, which is an indicator of minimal competence for purposes of licensure. 39 Among other things, Petitioner alleges, in paragraph 19 of the Rule Challenge Petition, that the Challenged Rule is invalid under section 120.52(8)(d) because it does not address circumstances where an examination cannot be administered due to force majeure. As maintain the exam's integrity. Additionally, the lack of guarantee of internet service reliability and functionality for every examinee is a crucial consideration, since failed internet connections could significantly affect the fairness of the exam.

Florida Laws (15) 120.52120.54120.541120.542120.56120.569120.57120.68455.217471.005471.007471.013471.01590.70190.702 Florida Administrative Code (8) 1-1.01061G15-18.01161G15-19.00161G15-20.001061G15-20.00261G15-21.00161G15-21.00461G15-30.002 DOAH Case (1) 20-3075RX
# 10

Can't find what you're looking for?

Post a free question on our public forum.
Ask a Question
Search for lawyers by practice areas.
Find a Lawyer